0% found this document useful (0 votes)
46 views

Question cardiology

Cardiology revision questions
Copyright
© © All Rights Reserved
We take content rights seriously. If you suspect this is your content, claim it here.
Available Formats
Download as DOCX, PDF, TXT or read online on Scribd
0% found this document useful (0 votes)
46 views

Question cardiology

Cardiology revision questions
Copyright
© © All Rights Reserved
We take content rights seriously. If you suspect this is your content, claim it here.
Available Formats
Download as DOCX, PDF, TXT or read online on Scribd
You are on page 1/ 15

Question 1.

Which one of the following statements about the jugular venous


pulse (JVP) is true?
A) The height reflects left atrial pressure
B) The JVP falls with abdominal compression
C) It is best examined with the patient supine
D) Large systolic waves are associated with tricuspid regurgitation
E) Prominent a waves suggest atrial fibrillation

Question 2. In the majority of individuals, the atrioventricular (AV) node is supplied by


the:
A) Left main stem artery
B) Left anterior descending artery
C) Circumflex artery
D) Right coronary artery
More on Coronary anatomy
E) Obtuse marginal artery

Question 3. A 57-year-old man presents with an acute myocardial infarction affecting


the anterior part of the septum and anterior wall of the left ventricle. Which of the
following vessels is most likely to have been occluded?
A) Left anterior descending artery
B) Circumflex artery
C) Right coronary artery
D) Right posterior descending artery
E) Obtuse marginal artery

Question 4. Which one of the following statements regarding the nerve supply of the
heart is correct?
A) Beta1-adrenoceptors mediate negatively chronotropic effects
B) Beta2-adrenoceptors mediate positively inotropic effects
C) Adrenergic fibres reach the heart via the vagus nerves
D) Cholinergic nerve fibres act on the sinoatrial node to slow heart rate
E) Parasympathetic nerve fibres act on muscle fibres in the atria and ventricles
to reduce the strength of myocardial contraction

Question 5. Which one of the following is not an appropriate indication for an exercise
(stress) ECG?
A) Evaluation of stable angina
B) Evaluation of unstable angina
C) Assessment of prognosis following a myocardial infarction
D) Assessment of outcome following coronary angioplasty
E) Diagnosis of exercise-induced arrhythmias

Question 6. Which of the following symptoms is most suggestive of myocardial


ischaemia?
A) Epigastric discomfort after eating
B) Exertional breathlessness and wheeze
C) Exertional breathlessness and throat tightness
D) Breathlessness with perioral and digital paraesthesia
E) Rapid palpitation on exertion

Question 7. A 58-year-old woman presents with dyspnoea and central chest and throat
tightness, which occur when walking her dog and force her to stop and rest. She has
smoked 20 cigarettes a day for the last 30 years. Examination is unremarkable apart
from some lesions around her eye:
What abnormality is likely to be found on blood tests?
A) Renal impairment
B) Hypercholesterolaemia
C) Hyperthyroidism
D) Hyperglycaemia
E) Hypocalcaemia

Question 8. The patient from the previous question (Q7) is found to have a normal 12-
lead ECG and chest X-ray. Which of the following would be the most appropriate next
investigation?
A) Coronary angiogram
B) Exercise ECG
C) Echocardiogram
D) 24-hour tape
E) Pulmonary function tests

Question 9. The patient from the previous two questions (Q7 and Q8) undergoes an
exercise ECG, which is positive but shows no high-risk features. Blood tests confirm a
raised plasma cholesterol of 7.2 and blood pressure is 158/92 mmHg. All of the
following are appropriate secondary prevention measures except one. Which is the
exception?
A) Aspirin
B) Ramipril
C) Spironolactone
D) Simvastatin
E) Smoking cessation

Question 10. Which of the following disorders is most likely to present with isolated
right heart failure?
A) Aortic stenosis
B) Mitral regurgitation
C) Systemic hypertension
D) Dilated cardiomyopathy
E) Recurrent pulmonary emboli

Question 11. Which of the following findings on clinical examination is most suggestive
of left heart failure over right heart failure?
A) Raised jugular venous pressure (JVP)
B) Hepatomegaly
C) Fine bi-basal crepitations
D) Peripheral oedema
E) Ascites

Question 12. A 68-year-old man presents with acute pulmonary oedema. Which one of
the following examination findings is most suggestive of pre-existing structural heart
disease?
A) Elevated jugular venous pressure
B) Basal lung crepitations
C) Displaced apex beat
D) Gallop rhythm
E) Cool, clammy peripheries

Question 13. Which of the following cardiac medications most commonly causes
peripheral oedema?
A) Digoxin
B) Amlodipine
C) Atenolol
D) Nicorandil
E) Ramipril

Question 14. A patient presents with severe bilateral lower limb pitting oedema. Which
of the following features would suggest a cause other than chronic cardiac failure?
A) Raised jugular venous pressure
B) Hepatomegaly
C) Peripheral muscle wasting
D) Hyponatraemia
E) Heavy proteinuria

Question 15. A 63-year-old man with ischaemic heart disease attends the cardiology
clinic with worsening symptoms of chronic heart failure. His treatment is modified. Three
weeks later, routine blood tests by the GP reveal a serum potassium of 6.5 mmol/l
(reference range 3.6-5.1). Which of the following drugs is the most likely culprit?
A) Furosemide
B) Ramipril
C) Digoxin
D) Bisoprolol
E) Aspirin

Question 16. Which one of the following treatments should be avoided in patients with
acute left ventricular failure?
A) Intravenous glyceryl trinitrate
B) Non-invasive positive pressure ventilation
C) Beta-blockers
D) Loop diuretics
E) Intravenous morphine

Question 17. Angiotensin receptor blockers (ARBs) are frequently used in patients
intolerant of ACE inhibitors. Which of the following adverse effects of ACE inhibitor
therapy is most likely to be improved by switching to an ARB?
A) Renal impairment
B) Postural hypotension
C) Dry cough
D) Hyperkalaemia
E) Hyponatraemia

Question 18. All of the following agents except one have been shown to reduce
mortality in patients with chronic heart failure. Which is the exception?
A) Angiotensin receptor blockers
B) ACE inhibitors
C) Beta-blockers
D) Loop diuretics
E) Spironolactone
Question 19. Which one of the following diagnoses is least likely to account for an
episode of loss of consciousness?
A) Carotid sinus syndrome
B) Transient ischaemic attack
C) Stokes-Adams attack
D) Ventricular tachycardia
E) Hypertrophic obstructive cardiomyopathy
Question 20. In differentiating cardiac syncope from seizures, which one of the
following most strongly suggests cardiac syncope?
A) No warning prior to blackout
B) Cyanosis during unconscious period
C) Urinary incontinence
D) Brief twitching during unconscious period
E) Rapid recovery (< 1 min) following the collapse
Question 21. A patient admitted to the coronary care unit with an acute anterior
myocardial infarction loses consciousness during the morning ward round. Below is a
rhythm strip taken from his continuous cardiac telemetry:
What is the most urgent priority in this patient's management?
A) IV amiodarone
B) External cardiac pacing
C) IV adrenaline (epinephrine)
D) Endotracheal intubation and ventilation
E) Defibrillation
Question 22. A 74-year-old man with severe pneumonia on the medical highdependency
unit suffers a cardiac arrest. The cardiac monitor shows asystole.
Cardiopulmonary resuscitation (CPR) is commenced and IV adrenaline (epinephrine) is
administered. Which one other drug should be given at this stage?
A) Amiodarone
B) Atropine
C) Lidocaine
D) Magnesium sulphate
E) Sodium bicarbonate
Question 23. Which of the following disorders characteristically produces a loud first
heart sound (S1)?
A) Atrial septal defect
B) Mitral stenosis
C) Aortic regurgitation
D) Left ventricular hypertrophy
E) Constrictive pericarditis
Question 24. Wide fixed splitting of the second heart sound occurs in which one of the
following disorders?
A) Patent ductus arteriosus
B) Aortic stenosis
C) Ventricular septal defect
D) Mitral regurgitation
E) Atrial septal defect
Question 25. A fourth heart sound is least likely to be heard in which of the following
conditions?
A) Systemic hypertension
B) Aortic stenosis
C) Hypertrophic cardiomyopathy
D) Atrial fibrillation
E) Ischaemic heart disease
Question 26. The combination of a third heart sound and soft first heart sound is most
characteristic of which of the following disorders?
A) Aortic regurgitation
B) Pulmonary stenosis
C) Mitral regurgitation
D) Atrial septal defect
E) Ventricular septal defect
Question 27. Which of the following conditions is most likely to produce a murmur in
early diastole?
A) Ventricular septal defect (VSD)
B) Mitral stenosis
C) Tricuspid regurgitation
D) Aortic regurgitation
E) Pulmonary stenosis
Question 28. Which one of the following not a cause of sinus tachycardia?
A) Anaemia
B) Raised intracranial pressure
C) Thyrotoxicosis
D) Phaeochromocytoma
E) Nebulised salbutamol
Question 29. A 58-year-old woman presents to the hospital Accident and Emergency
department with sudden onset of palpitations. Below is a rhythm strip of her ECG
before and after carotid sinus massage:
What is the cause of her palpitations?
A) Atrial fibrillation
B) Atrial tachycardia
C) Atrial flutter
D) Atrioventricular (AV) nodal re-entry tachycardia
E) Sinus tachycardia
Question 30. A 47-year-old man presents to the hospital Accident and Emergency
department with palpitations and is found to be in atrial fibrillation (AF) with a rapid
ventricular rate. He cardioverts spontaneously but this is his third such episode in the
last 2 years. He is otherwise well and drinks no alcohol. Blood pressure is 152/88
mmHg. Routine blood tests are normal and an echocardiogram reveals no structural
heart disease. Which of the following medications would be the most appropriate firstline
therapy to prevent further episodes?
A) Bisoprolol
B) Verapamil
C) Ramipril
D) Digoxin
E) Amiodarone
Question 31. Which one of the following is a contraindication to DC cardioversion for
persistent atrial fibrillation?
A) Age over 75
B) Atrial fibrillation present for > 1 year
C) Moderate to severe left ventricular impairment
D) Subtherapeutic INR
E) Relapse after two previous cardioversions
Question 32. Which one of the following medications would not be suitable for rate
control in persistent atrial fibrillation?
A) Digoxin
B) Atenolol
C) Verapamil
D) Diltiazem
E) Amlodipine
Question 33. Intravenous adenosine is most likely to restore sinus rhythm in which of
the following tachyarrhythmias?
A) Atrial flutter
B) Atrial fibrillation
C) Atrioventricular (AV) nodal re-entrant tachycardia
D) Atrial tachycardia
E) Ventricular tachycardia
Question 34. A 24-year-old woman presents to her GP with recurrent episodes of
palpitation. She describes several episodes where she has suddenly felt her heart
beating very rapidly. They last anywhere from a few minutes to an hour before selfterminating.
She feels very frightened during these episodes and is worried she may
have a serious heart condition. Examination is normal and she appears to be in sinus
rhythm. A segment of her 12-lead ECG is shown below.
What is the likely diagnosis?
A) Paroxysmal atrial fibrillation
B) Panic attacks
C) Wolff-Parkinson-White syndrome
D) Ventricular ectopic beats
E) Recurrent ventricular tachycardia
Question 35. The patient in the previous question (Q34) finds these episodes of
palpitation distressing and asks about treatment to prevent further episodes. Which of
the following is the treatment of choice?
A) Digoxin
B) Dual chamber pacemaker
C) Verapamil
D) Catheter ablation of accessory pathway
E) Amiodarone
Question 36. A 55-year-old man is brought to the emergency department by
ambulance after collapsing at work. His ECG shows a broad complex tachycardia:
Which of the following features most strongly favours a diagnosis of ventricular
tachycardia (VT) over supraventricular tachycardia with bundle branch block?
A) History of ischaemic heart disease
B) Termination with IV adenosine
C) Presence of capture and fusion beats on ECG
D) QRS duration of 150 ms
E) Right axis deviation on ECG
Question 37. The patient in the previous question (Q36) is diagnosed with ventricular
tachycardia (VT). He is aware of his heart racing but is otherwise asymptomatic. Heart
rate is 170 bpm. Blood pressure is 110/70 mmHg. There is no evidence of pulmonary
oedema. What is the most appropriate next treatment?
A) IV metoprolol
B) IV amiodarone
C) IV flecainide
D) DC cardioversion
E) Overdrive pacing
Question 38. Below is an ECG recording from a patient on the coronary care unit, who
is on continuous telemetry. She had been admitted the previous day with an episode of
syncope:
What is the diagnosis?
A) Idioventricular rhythm ('slow ventricular tachycardia')
B) Ventricular fibrillation
C) Torsades de pointes
D) Supraventricular tachycardia with aberrant conduction
E) Sick sinus syndrome
Question 39. What abnormality is most likely to be seen on a 12-lead ECG from the
patient in the previous question (Q38) when she was in sinus rhythm?
A) ST elevation
B) Prolonged PR interval
C) Broad QRS complexes
D) Prolonged QT interval
E) Delta wave
Question 40. Which of the following drugs is most likely to cause a prolonged QT
interval?
A) Omeprazole
B) Digoxin
C) Clarithromycin
D) Diazepam
E) Morphine
Question 41. Which of the following drug treatments is most appropriate for a patient
with torsades de pointes?
A) Magnesium
B) Amiodarone
C) Lidocaine
D) Digoxin
E) Sotalol
Question 42. A 77-year-old patient is referred to the cardiology outpatient clinic with
recurrent dizzy spells, and a 24-hour tape is performed. The ECG strip below is
recorded during a typical episode of dizziness:
What rhythm abnormality is shown?
A) First-degree atrioventricular (AV) block
B) Mobitz type I second-degree AV block
C) Mobitz type II second-degree AV block
D) Third-degree (complete) heart block
E) Junctional rhythm
Question 43. What is the most appropriate management of the patient in the previous
question (Q42)?
A) Review in 6 months and consider treatment if any syncopal episodes
B) Start amiodarone and repeat 24-hour tape
C) Atrial pacemaker
D) Dual-chamber pacemaker
E) Implantable cardiac defibrillator
Question 44. Which one of the following is not a manifestation of digoxin toxicity?
A) Nephrotoxicity
B) Altered colour vision
C) Bradycardia
D) Vomiting
E) Ventricular tachycardia
Question 45. Which of the following features renders an atherosclerotic plaque most
vulnerable to rupture?
A) Thick fibrous cap
B) Calcification
C) Lipid-rich core
D) Extensive collagenous cross-struts
E) High concentration of smooth muscle cells
Question 46. Which one of the following is not an independent risk factor for
atherosclerosis?
A) Systolic hypertension
B) Raised high-density lipoprotein (HDL) cholesterol
C) Hyperhomocysteinaemia
D) Central obesity
E) Diabetes mellitus
Question 47. A 60-year-old man is referred to the cardiology outpatient department
with a 6-month history of exertional chest pain relieved by rest and glyceryl trinitrate
(GTN) spray. As part of his assessment he undergoes an exercise ECG. Which of the
following features on exercise ECG would be the strongest indication for proceeding to
coronary angiography?
A) Rise in systolic blood pressure of 60 mmHg during test
B) Widespread 1-2 mm upward-sloping ST depression during stage 2 of the
Bruce protocol without symptoms
C) Failure to complete stage 1 of the Bruce protocol due to hip pain
D) Chest pain and 2-3 mm horizontal ST depression in lateral leads during
stage 4 of the Bruce protocol
E) Chest pain and 1-2 mm downward-sloping ST depression in inferior
leads during stage 1 of the Bruce protocol
Question 48. Which one of the following drugs is not an anti-anginal agent?
A) Clopidogrel
B) Atenolol
C) Diltiazem
D) Nicorandil
E) Isosorbide mononitrate
Question 49. Which one of the following features is least consistent with a diagnosis of
unstable angina?
A) Chest pain on minimal exertion
B) Persistent ST segment elevation on ECG
C) No preceding history of stable angina
D) More than two episodes of chest pain at rest in the last 24 hours
E) T wave inversion on ECG
Question 50. A 68-year-old man with a long history of angina presents with a
worsening of anginal symptoms. He was previously able to walk up to 1 mile before the
onset of symptoms but for the past few days has been experiencing pain after just 50
yards and, on one occasion, at rest. Which of the following factors would have the least
impact on his short-term risk of death or acute myocardial infarction?
A) Recurrent pain at rest
B) ST segment depression on ECG
C) Elevated serum cholesterol
D) Persistent deep T-wave inversion on ECG
E) Elevated serum troponin
Question 51. The patient from the previous question (Q50) is admitted to hospital and
an admission ECG reveals 1 mm ST depression in leads V5 and V6. In the absence of
any contraindications, which of the following would be the optimal combination of
antiplatelet agents and anticoagulants to prescribe?
A) Aspirin alone
B) Aspirin and low molecular weight heparin
C) Aspirin and clopidogrel
D) Aspirin, clopidogrel and low molecular weight heparin
E) Aspirin, clopidogrel and warfarin
Question 52. After 48 hours of optimal medical treatment, the patient from the previous
question (Q51) is still experiencing occasional short-lived episodes of chest pain at rest.
His ECGs while he is pain-free are normal, but those taken during pain show
recurrence of ST segment depression in the lateral leads. Troponin T is modestly
elevated. What would be the best investigation to perform next in this patient?
A) 24-hour ECG
B) Exercise ECG
C) Echocardiogram
D) Myocardial perfusion scan
E) Coronary angiogram
Question 53. A 48-year-old man presents with a 6-hour history of severe central chest
pain. He is unable to give much history but declares that the pain is the worst he has
ever felt. He was previously well with no history of cardiovascular disease. On
examination he appears pale, sweaty and distressed. His ECG is shown below:
What is the diagnosis?
A) Aortic dissection
B) Acute anterior myocardial infarction
C) Acute inferior myocardial infarction
D) Unstable angina
E) Pericarditis
Question 54. The patient from the previous question (Q53) with acute anterior ST
elevation myocardial infarction (MI) is treated immediately with IV morphine, sublingual
glyceryl trinitrate and aspirin. He appears less distressed but his heart rate is 118 bpm
and blood pressure is 195/115 mmHg. Heart sounds 1 and 2 are present with no added
sounds or murmurs and the chest is clear to auscultation. Which of the following
options would be the +best next step in his management?
A) Thrombolysis
B) IV magnesium
C) Sublingual nifedipine
D) IV metoprolol
E) Oral ramipril
Question 55. A 62-year-old woman presents with a 2-hour history of severe chest pain
and has the following ECG:
While awaiting thrombolysis, she complains of lightheadedness. Her pulse rate is 38
beats per minute and telemetry shows sinus bradycardia. What is the most appropriate
next treatment?
A) Insertion of temporary pacing wire
B) IV atropine
C) External cardiac pacing
D) IV isoprenaline
E) IV atenolol
Question 56. Which one of the following long-term treatments does not confer
prognostic benefit in survivors of acute myocardial infarction (MI)?
A) Aspirin
B) Angiotensin-converting enzyme (ACE) inhibitor
C) Beta-blocker
D) Statin
E) Isosorbide mononitrate
Question 57. In a patient with peripheral arterial disease, which one of the following
features suggests the worst prognosis?
A) Claudication distance of 50-100 metres
B) Night pain
C) Diminished pedal pulses
D) Ankle:brachial pressure index (ABPI) ratio of 0.8
E) Bruit over femoral artery
Question 58. In a patient presenting with acute lower limb ischaemia, which one of the
following clinical features would favour a diagnosis of embolism over thrombosis in
situ?
A) Onset of symptoms over hours
B) History of intermittent claudication
C) Previously undiagnosed atrial fibrillation
D) Absent contralateral lower limb pulses
E) Presence of bruits
Question 59. Which one of the following clinical features is least consistent with a type
B aortic dissection?
A) Severe tearing chest pain
B) Elevated blood pressure
C) Asymmetry of femoral pulses
D) New early diastolic murmur
E) Acute renal failure
Question 60. Which of the following is not a recognised risk factor for essential
hypertension?
A) Hyperlipidaemia
B) Obesity
C) High salt intake
D) Intrauterine growth retardation
E) Alcohol excess
Question 61. The following endocrine diseases are all recognised causes of secondary
hypertension except one. Which is the exception?
A) Acromegaly
B) Congenital adrenal hyperplasia
C) Conn's syndrome
D) Phaeochromocytoma
E) Addison's disease
Question 62. A 77-year-old woman in good health is found by her general practitioner
to have persistently elevated blood pressure, with a typical reading of 162/84 mmHg.
Which of the following agents would be the most appropriate initial treatment?
A) Angiotensin-converting enzyme (ACE) inhibitor
B) Angiotensin receptor blocker
C) Beta-blocker
D) Thiazide diuretic
E) Alpha-blocker
Question 63. A 53-year-old male with hypertension has been taking 10 mg of ramipril
for several months. Despite this, his blood pressure control remains suboptimal. There
are no compliance issues. Which one of the following agents would be the best add-on
therapy?
A) Angiotensin receptor blocker
B) Beta-blocker
C) Calcium channel antagonist
D) Hydralazine
E) Alpha-blocker
Question 64. A 19-year-old man presents with recent onset of breathlessness and
sharp, central chest pain exacerbated by movement and coughing. His heart rate is 110
beats per minute. On auscultation there is a soft pansystolic murmur and a pericardial
friction rub. Echocardiography demonstrates mitral regurgitation. Antistreptolysin O
antibody titres (ASOT) are 500 U/ml (normal range < 200). What is the likeliest
diagnosis?
A) Infective endocarditis
B) Viral myocarditis
C) Acute rheumatic fever
D) Viral pericarditis
E) Dressler's syndrome
Question 65. In the patient from the previous question (Q64), which one of the
following features would clinch the diagnosis of rheumatic fever?
A) Temperature > 38oC
B) Positive throat swab culture
C) Cardiac dilatation on echocardiography
D) First-degree block on ECG
E) Flitting polyarthritis
Question 66. A 53-year-old lady complains of progressively worsening exertional
dyspnoea associated with deterioration in exercise tolerance over the past 2 years. On
examination her pulse is irregularly irregular and of small volume. There is a lowpitched
mid-diastolic murmur audible at her apex. What is the likeliest cause of her
breathlessness?
A) Dilated cardiomyopathy
B) Ischaemic heart disease
C) Ventricular septal defect
D) Mitral stenosis
E) Aortic regurgitation
Question 67. With reference to the patient from the previous question (Q66) who has
mitral stenosis, which one of the following features would suggest that her condition has
been complicated by pulmonary hypertension?
A) Loud first heart sound (S1)
B) Opening snap
C) Pansystolic murmur radiating to the axilla
D) Tapping apex beat
E) Loud pulmonary component of second heart sound (P2)
Question 68. Which one of the following would be a contraindication to mitral balloon
valvuloplasty?
A) Mitral valve orifice < 2 cm2
B) Concomitant mitral regurgitation
C) Mobile valvular apparatus
D) Atrial fibrillation
E) Enlarged left atrium
Question 69. The following are all recognised causes of mitral regurgitation except
one. Which is the exception?
A) Papillary muscle rupture
B) Dilated cardiomyopathy
C) Acute rheumatic fever
D) Subacute bacterial endocarditis
E) Congenitally bicuspid mitral valve
Question 70. Which one of the following features is likely to be found in mitral
regurgitation but not in mitral stenosis?
A) Irregularly irregular pulse
B) Third heart sound (S3)
C) Right ventricular heave
D) Accentuated pulmonary component of second heart sound
E) Bi-basal crepitations
Question 71. A 64-year-old man is found to have an ejection systolic murmur during a
routine medical examination. Which of the following associated clinical signs would lend
most support to a diagnosis of aortic stenosis?
A) Displaced apex beat
B) Loud aortic component of second heart sound (A2)
C) Louder in inspiration
D) Wide pulse pressure
E) Slow-rising carotid pulse
Question 72. A patient with known aortic stenosis is followed up annually in the
cardiology outpatient clinic. Which of the following would be the strongest indication for
aortic valve replacement?
A) Syncopal episodes
B) Prolonged murmur and thrusting apex beat on examination
C) Left bundle branch block on ECG
D) Reduced valve area on echocardiography
E) High pressure gradient between left ventricle and aorta at cardiac
catheterisation
Question 73. You are asked to examine a dyspnoeic patient on the cardiology ward.
She has bounding peripheral pulses, a blood pressure of 190/70 mmHg, a displaced,
heaving apex beat and a loud early diastolic murmur. Which of the following underlying
pathologies would be least likely to account for these physical findings?
A) Rheumatic heart disease
B) Acute aortic dissection
C) Syphilitic aortitis
D) Marfan's syndrome
E) Ankylosing spondylitis
Question 74. A patient with a history of long-standing rheumatic mitral stenosis
presents with a 6-week history of malaise, drenching sweats and lethargy. On
examination she has poor dentition, a temperature of 38.5°C and a loud pansystolic
murmur at apex radiating to axilla. You suspect a diagnosis of subacute bacterial
endocarditis. Which of the following microorganisms is the likeliest culprit?
A) Viridans streptococci
B) Staphylococcus aureus
C) Coxiella burnetti
D) Coagulase-negative staphylococci
E) Haemophilus
Question 75. Echocardiography confirms the presence of a vegetation on the mitral
valve and mitral regurgitation. Six sets of blood cultures are sent. While awaiting
results, which of the following antibiotic regimens would be the most appropriate
empirical treatment?
A) Flucloxacillin
B) Benzyl penicillin
C) Gentamicin
D) Benzyl penicillin and gentamicin
E) Vancomycin and flucloxacillin
Question 76. According to the modified Duke criteria, which one of the following is
considered a major criterion for the diagnosis of infective endocarditis?
A) Pyrexia > 38°C
B) Pre-existing aortic stenosis
C) Splinter haemorrhages
D) Vegetations on echocardiography
E) Osler's nodes
Question 77. A continuous machinery murmur, loudest over the left second intercostal
space, is associated with which of the following congenital cardiac abnormalities?
A) Atrial septal defect
B) Ventricular septal defect
C) Patent ductus arteriosus
D) Tricuspid atresia
E) Pulmonary stenosis
Question 78. An 18-year-old boy, who previously attended the paediatric cardiology
clinic but defaulted from follow-up after his 14th birthday, now presents to his GP with
increasingly severe breathlessness, fatigue and reduced exercise tolerance. On
examination he is centrally cyanosed with clubbing of the fingers and toes. Which of the
following is the most likely underlying congenital cardiac abnormality?
A) Ventricular septal defect
B) Coarctation of the aorta
C) Pulmonary stenosis
D) Bicuspid aortic valve
E) Patent foramen ovale
Question 79. Which one of the following does not constitute a component of Fallot's
tetralogy?
A) Pulmonary stenosis
B) Ventricular septal defect
C) Overriding aorta
D) Patent ductus arteriosus
E) Right ventricular hypertrophy
Question 80. A patient who immigrated to the UK from West Africa 18 months ago
presents with progressive ankle swelling and abdominal distension. On examination he
has marked bilateral lower limb oedema, a palpably enlarged liver and ascites. His
jugular venous pulse is elevated and rises during inspiration. Lung fields are clear to
auscultation. His chest X-ray is shown below:
Which of the following is the likeliest underlying aetiology?
A) Tuberculosis
B) Trypanosomiasis
C) Syphilis
D) Malaria
E) Haemochromatosis

You might also like